¿Violación de la segunda ley de la termodinámica? [cerrado]

Aquí hay una máquina que parece violar la segunda ley de la termodinámica:2 elipsoides confocales y un anillo

  • A y B son cuerpos puntuales negros de la misma temperatura ( inicialmente ).
  • todo es rotacionalmente simétrico alrededor del eje A B
  • mi y F son elipsoides con focos A y B , hecho de un material reflectante
  • C D y mi F son secciones de un anillo reflexivo
  • no hay aire

El estado estable de la máquina es A tener una temperatura más alta que B porque

  • El calor irradiado por B todo es absorbido por A (a través de caminos B k A y B j A ).
  • El calor irradiado por A es absorbido por B (a través de caminos A k B y A j B ) O por A (a través de caminos A GRAMO I A )

Esto parece violar la segunda ley.

Entonces, ¿dónde está el agujero aquí?

PD. Si bien los cuerpos puntuales y los espejos perfectos no existen, tenga en cuenta que tenemos bastante margen aquí: un elipsoide izquierdo enorme y un elipsoide derecho pequeño conducirán a casi el 50% de toda la radiación de A reflejando de nuevo a A . Por lo tanto, los cuerpos "pequeños" y los espejos con un 90% de eficiencia deberían estar bien.

¿Por qué votar negativo? ¡Por favor explique!
Los espejos perfectos y los cuerpos negros puntuales no son físicos. Sin embargo, a su punto, puede haber una razón más "fundamental". Alguien más inteligente que yo tendrá que abordar eso.
@garyp: por favor vea PD
¿ Podría explicar explícitamente por qué esto contradiría la segunda ley ? Sin juegos de palabras, por favor, calcule las entropías que afirma que están violando la ley, es decir, ¿dónde disminuye la entropía aquí ? (Tenga en cuenta que, aunque haya encontrado un estado estable, no se garantiza que ninguna configuración inicial llegue realmente a ese estado)
@ACuriousMind: si me equivoco y la creación de una diferencia de temperatura no viola la segunda ley, ¡haga el cálculo en una respuesta y lo aceptaré con gusto! TIA.
Hay una diferencia de temperatura. ¿Así que lo que? El interior de un termo "perfecto" bien puede tener una temperatura diferente a la del exterior durante tiempos infinitamente largos, pero eso no contradice nada, por lo que puedo ver. La segunda ley solo establece que la entropía no disminuye .
@ACuriousMind: aquí la situación es muy diferente a la de un termo: la diferencia de temperatura aumenta .
@ACuriousMind: los estados de equilibrio siempre se caracterizan por una temperatura uniforme en la termodinámica, vea todas las referencias para la afirmación "La temperatura dentro de un sistema en equilibrio termodinámico es uniforme en el espacio y en el tiempo" en la sección "temperatura uniforme" del " Equilibrio termodinámico" artículo wiki
@ACuriousMind: con respecto al ejemplo de un termo perfecto, supongo que el término "sistema" termodinámico presupone que existe al menos alguna posibilidad de interacción térmica entre todas las partes, sin importar cuán pequeñas o raras sean, en este sentido un perfecto En realidad, el termo no podría considerarse solo como el caso límite de una serie de termos cada vez mejores, ya que en cada miembro bueno pero no perfecto de la serie, existe al menos alguna posibilidad de transferencia de calor que permite que la temperatura se vuelva uniforme en el largo plazo
Votación para cerrar. Esta es otra variante más de la paradoja del elipsoide en termodinámica. La resolución es simple: estás asumiendo partículas puntuales. Las fuentes no puntuales bañarán de luz toda la estructura. Véase, por ejemplo, Yoder & Adkins, "Resolución de la paradoja del elipsoide en termodinámica". Revista estadounidense de física 79.8 (2011): 811-818. .
@DavidHammen: entonces, la respuesta proporcionada es realmente correcta, lo único que falta es la referencia que proporcionó. ¡Gracias!
¿Votó para cerrar su propia pregunta como un duplicado ahora que ha buscado un poco más profundo? Estoy impresionado. (En serio. Estoy impresionado. Eso demuestra carácter.)
@DavidHammen: ¡gracias! Pero, ¿por qué la pregunta se cierra como "poco clara" y no como "duplicada"? ¡Creo que la pregunta es un engaño perfectamente claro!
@sds - Votamos sobre diferentes cosas. La última vez que revisé, hubo un voto por duplicado, dos por fuera de tema, uno por poco claro. La última persona en votar debe haber elegido incierto, por lo que hay un empate entre "incierto" y "fuera de tema". El último voto gana el desempate. ¿Tu pregunta fue "poco clara"? De nada. Me quedó bastante claro. Voté "fuera de tema" porque la segunda ley de la termodinámica se mantendrá mucho después de que QM y GR sean reemplazados por algo aún mejor.
@DavidHammen: No veo cómo la permanencia de la segunda ley tiene alguna relación con la cuestión de estar dentro/fuera del tema aquí. En cualquier caso, creo que cerrarlo como un engaño tendría más sentido, y haría que el enlace a la otra pregunta fuera más prominente (me dicen que evite discusiones prolongadas en los comentarios, así que supongo que lo revisaré ahora) . Gracias de nuevo.
Este sitio se centra en la física convencional. Las preguntas relacionadas con las violaciones de la segunda ley de la termodinámica generalmente están fuera de tema porque van en gran medida en contra de la corriente principal.
@David Hammen: hacer una pregunta sobre cómo una configuración evita violar alguna ley no es lo mismo que decir con confianza que viola esa ley, supuse que la pregunta era sincera y no una forma retórica de decir "mira, he violado el ¡Segunda ley!" Los libros de texto de física a menudo presentan preguntas "rompecabezas" como estas con el fin de ayudar a la comprensión.
El lado grande podría contener más calor para la misma temperatura solo porque es más grande. Se enfoca en A, pero eso no significa que todo el calor va a A. En cambio, toda la cámara se calentaría a la misma temperatura que la cámara pequeña, luego ese calor adicional se derramaría a la cámara pequeña haciéndolas temperaturas iguales. Sé que las cámaras son vacíos, pero estoy hablando de calor por convección en forma de radiación electromagnética. ¿Me oyes en eso?

Respuestas (2)

Ingenioso. A y B son pequeños, pero no pueden ser puntos. La imagen de B se magnifica en A. Por lo tanto, si A y B tienen el mismo tamaño, parte de la luz de B perderá A.

¿Por qué "La imagen de B se magnifica en A"?
¿Puedes realmente hacer el cálculo con un radio finito? R para los cuerpos negros y comprobar que la transferencia de energía se equilibra? ¿Qué pasa con el límite R 0 ?
@sds: eso se basa en la óptica del espejo: los espejos parabólicos cóncavos crean imágenes virtuales ampliadas de objetos en cualquier posición más cercana que el centro de curvatura del espejo (que tiene el doble de la distancia focal, por lo que su A y B estarían en este rango) , vea aquí una foto de lo que realmente se ve en uno, y un diagrama de rayos, vea la ilustración inferior izquierda en la parte superior de la pág. 332 en el pdf aquí (el objeto es la flecha azul, la imagen virtual es la flecha naranja)
@StevenMathey En el límite que R 0 La Ley de Stefan-Boltzmann hace que la potencia emitida tienda a 0. Dado que esta es una ley universal para todos los cuerpos negros, no hay ningún parámetro que pueda ajustar para mantener la respuesta finita en el límite del área de superficie cero.
@StevenMathey: creo que un cálculo detallado probablemente sería muy laborioso, ya que debe tener en cuenta no solo las imágenes de A y B en cada sección parabólica y la sección plana, sino también el efecto de salón de espejos donde algunos de la luz de la imagen de B del espejo cercano no alcanza a A y, en cambio, se refleja en el espejo cercano a A, creando otra imagen cuyo tamaño es diferente al de la imagen de B creada cuando la luz de B toma un camino directo a el espejo cerca de A. Sin embargo, apostaría a que la respuesta de akrasia es la clave básica para la solución.
En realidad, tal vez un enfoque más simple sería comenzar con la suposición de un equilibrio en el que no solo A y B tienen la misma temperatura, sino que cada punto en los espejos brilla con una distribución de fotones de cuerpo negro a esta temperatura, con todas las superficies creando un gas fotónico a esa temperatura en el espacio intermedio... entonces tal vez podrías demostrar que en cada punto de cada superficie, la energía en cada frecuencia que llega a ese punto desde otros puntos equilibra exactamente la energía en cada frecuencia que asumimos el punto irradiaría.

Colóquese en la superficie del cuerpo A y dispare un rayo en cualquier dirección. Después de un cierto número de reflejos de las superficies del espejo (posiblemente ninguno), golpeará el cuerpo A o el cuerpo B. Ahora expanda el rayo en un cono muy estrecho, de modo que todos los rayos en el cono terminen en el mismo cuerpo. Mirando dentro de ese cono verás una intensidad radiante característica de la temperatura de ese cuerpo. Esto es cierto independientemente de cualquier efecto de aumento (una consecuencia del teorema de reciprocidad para los factores de vista). Y como lo mismo es cierto para cualquier rayo inicial, la intensidad radiante será uniforme en todas las direcciones y característica de la temperatura de ambos cuerpos. Por lo tanto, no habrá transferencia neta de calor.

No entiendo bien esto, ¿puede explicar en qué parte del argumento hizo uso de la suposición de que todo el sistema ha alcanzado el equilibrio? Si los dos cuerpos comienzan a diferentes temperaturas y ambos emiten radiación de cuerpo negro, no puede ser cierto al principio que "no habrá transferencia neta de calor", por lo que el argumento es erróneo o hay algún paso en el argumento de que hace uso de una suposición que es válida si suponemos que todo el sistema ya está en equilibrio, pero que no es válida en caso contrario.